3
$\begingroup$

Let $(R, \mathfrak{m})$ be a commutative Noetherian local ring and $M$ a finitely generated $R$-module. Let $x_1,...,x_t$ be an $M$-regular sequence and $I = (x_1,...,x_t)$. Is it true that $$\mathrm{Tor}_1^R(R/I^n, M) = 0$$ for all $n \geq 1$?

$\endgroup$
5
  • 1
    $\begingroup$ It's true for $n=1$ if the sequence is also $R$-regular, since then $\mathrm{Tor}_{1}^{R}(R/I,M) \simeq \mathrm{Tor}_{1}^{R/I}(R/I,M/I)$. (see Lemma 18.2.iii in Matsumura's CRT.) $\endgroup$ Jan 9, 2013 at 19:11
  • 1
    $\begingroup$ Lemma 18.2 in Matsumura need $x$ is both $R$-regular and $M$-regular. $\endgroup$ Jan 10, 2013 at 2:41
  • $\begingroup$ So Lemma 18.2 applies at least when $M$ has finite projective dimension. $\endgroup$ Jan 10, 2013 at 4:22
  • $\begingroup$ $I^{n-1}/I^n$ is a free $R/I$-module, so the statement follows from a simple induction. $\endgroup$
    – Angelo
    Jul 25, 2013 at 5:41
  • $\begingroup$ I wonder if this is true even in the $n=1$ case? I know it is true if $t=1$, but I wonder if it would be true when $n=1$ and $t>1$? $\endgroup$
    – user521337
    Aug 12, 2021 at 9:13

1 Answer 1

2
$\begingroup$

I've posted a proof here for the special case when $M$ is cyclic. Furthermore, I've mentioned that the result holds for finitely generated modules when the sequence is $R$-regular and $M$-regular.

$\endgroup$
1
  • $\begingroup$ Very nice proof. Thanks you very much YACP. The case $M$ is cyclic is an exercise in the book about tight closure of C. Huneke. Your proof even better than its solution (in my opinion). $\endgroup$ Jan 12, 2013 at 15:43

Your Answer

By clicking “Post Your Answer”, you agree to our terms of service and acknowledge you have read our privacy policy.

Not the answer you're looking for? Browse other questions tagged or ask your own question.